Difference between revisions of "2022 SSMO Team Round Problems/Problem 10"
Rounak iitr (talk | contribs) (→Solution) |
Rounak iitr (talk | contribs) (→Solution) |
||
Line 3: | Line 3: | ||
==Solution== | ==Solution== | ||
− | By Vieta's relation we get, <math>\sum_{cyc}{}\alpha=2</math> <math>\sum_{cyc}{}\alpha\beta=0</math> and <math>\prod_{cyc}{}\alpha=4 | + | By Vieta's relation we get, <math>\sum_{cyc}{}\alpha=2</math> <math>\sum_{cyc}{}\alpha\beta=0</math> and <math>\prod_{cyc}{}\alpha=4</math> |
Therefore we have to find the value of <cmath>\prod_{cyc}{}(\alpha^3+\beta\gamma)=\prod_{cyc}{}\left(\alpha^3+\frac{4}{\alpha}\right)=\prod_{cyc}{}\left(\frac{\alpha^4+4}{\alpha}\right)</cmath> | Therefore we have to find the value of <cmath>\prod_{cyc}{}(\alpha^3+\beta\gamma)=\prod_{cyc}{}\left(\alpha^3+\frac{4}{\alpha}\right)=\prod_{cyc}{}\left(\frac{\alpha^4+4}{\alpha}\right)</cmath> | ||
<cmath>\prod_{cyc}{}\left(\frac{\alpha^4+4}{\alpha}\right)=\frac{\prod_{cyc}{}(\alpha-(1+i))(\alpha-(1-i))(\alpha+(1-i))(\alpha+(1+i))}{\alpha\beta\gamma}</cmath> | <cmath>\prod_{cyc}{}\left(\frac{\alpha^4+4}{\alpha}\right)=\frac{\prod_{cyc}{}(\alpha-(1+i))(\alpha-(1-i))(\alpha+(1-i))(\alpha+(1+i))}{\alpha\beta\gamma}</cmath> |
Revision as of 10:32, 25 December 2023
Problem
If are the roots of the polynomial , find
Solution
By Vieta's relation we get, and
Therefore we have to find the value of